Arthritis & Surgery Flashcards

1
Q
  1. During reporting of a series of GP plain film, a chest radiograph shows superior rib notching. Which of the following is a cause of superior rather than inferior rib notching?

A. Aortic thrombosis

B. Subclavian obstruction

C. Pulmonary AVM

D. Rheumatoid arthritis

E. Superior Vena cava (SVC) obstruction

A

D. Rheumatoid arthritis

Other causes of superior rib notching include SLE, scleroderma, Sjögren’s syndrome, hyperparathyroidism, Marfan’s syndrome and osteogenesis imperfecta.

NF can cause superior and inferior rib notching.

How well did you know this?
1
Not at all
2
3
4
5
Perfectly
2
Q
  1. In a 50-year-old man with joint pains, involvement of which of the following indicates ochranosis rather than calcium pyrophosphate dihydrate crystal deposition disease (CPDD)?

A. Annulus fibrosis of lumbar intervertebral discs

B. Nucleus pulposis of the lumbar intervertebral discs

C. Sacroiliac joints

D. Triangular fibrocartilage in the distal radioulnar joints

E. Glenohumeral joint

A

B. Nucleus pulposis of the lumbar intervertebral discs

CPDD never involves nucleus pulposis, unlike ochranosis.

How well did you know this?
1
Not at all
2
3
4
5
Perfectly
3
Q
  1. Which of the following indicates grade 4 chondromalacia patellae on T1 MRI?

A. Loss of sharp dark margin between articular cartilage of the patella and trochlea

B. Focal hypointense areas extending to the articular surface but not down to the osseous surface

C. Focal hypointense areas extending from subchondral bone to cartilage surface

D. Focal hypointense areas extending to the cartilage surface with preservation of sharp cartilage margins

E. Focal hypointense areas not extending to cartilage surface

A

C. Focal hypointense areas extending from subchondral bone to cartilage surface

Grade 1: focal hypointense areas not extending to cartilage surface

Grade 2: focal hypointense areas extending to the cartilage surface with preservation of sharp cartilage margins

Grade 3: loss of sharp dark margin between articular cartilage of the patella and trochlea and focal hypointense areas extending to the articular surface but not down to the osseous surface.

How well did you know this?
1
Not at all
2
3
4
5
Perfectly
4
Q
  1. Which is the most characteristic feature in haemachromatosis?

A. Chondrocalcinosis

B. Small subchondral cysts with a fine rim of sclerosis

C. Symmetric joint space narrowing

D. Generalised osteopenia

E. Hook-like osteophytes on the radial aspects of the metacarpal heads

A

E. Hook-like osteophytes on the radial aspects of the metacarpal heads

Although all of the answers are features, hook-like osteopohytes are the most characteristic. Arthropathy is seen in 50%, and chrondrocalinosis in more than 60%, with knees being most commonly affected.

How well did you know this?
1
Not at all
2
3
4
5
Perfectly
5
Q

@# 29. Considering the imaging features of extraarticular Pigmented Villonodular Synovitis (PVNS):

A. Typically manifests as a soft tissue mass in 20% of cases

B. Osseous abnormalities are present in the vast majority

C. Extrinsic erosion is the most common osseous abnormality

D. Radiographs are normal in 80% of cases

E. Extensive erosions are more common in the knee

A

C. Extrinsic erosion is the most common osseous abnormality

Manifests as a soft tissue mass in 50-70% of cases, with normal radiographs in 20% and osseous abnormalities in 5-25%.

The most common osseous abnormality are extrinsic erosions which are more often present in the ankle and foot.

How well did you know this?
1
Not at all
2
3
4
5
Perfectly
6
Q
  1. Which of the following favours a diagnosis of rheumatoid arthritis rather than tuberculous arthritis?

A. Periarticular osteopenia

B. Marginal erosion

C. Relatively late sparing of joint space

D. Joint effusion

E. Uneven and thick synovial proliferation

A

E. Uneven and thick synovial proliferation

The remaining answers are also seen in TB arthritis. Even and thin synovium, large bone erosions, rim enhancement around bone erosion and extra-articular cystic masses are more frequently seen in TB arthritis.

How well did you know this?
1
Not at all
2
3
4
5
Perfectly
7
Q
  1. In a 50-year-old woman with arthralgia, which of the following favours rheumatoid arthritis over Systemic Lupus Erythematosus (SLE)?

A. Prominent subluxation of metacarpal phalyngeal joints (MCPJ)

B. Usually bilateral and symmetrical

C. Absence of erosion

D. Radiographically similar to Jaccoud’s arthropathy

E. Hyperextension of Distal Interphalyngeal (DIP) and flexion of Proximal Interphalyngeal (PIP) joints

A

E. Hyperextension of Distal Interphalyngeal (DIP) and flexion of Proximal Interphalyngeal (PIP) joints

A-D are features seen in SLE.

How well did you know this?
1
Not at all
2
3
4
5
Perfectly
8
Q
  1. Ivory phalanx, tuft resorption, and pencil-in-cup deformity of the middle phalynx in a patient with asymmetric arthritis are most likely to represent:

A. Psoriatic arthritis

B. Haemachromatosis

C. Rheumatoid arthritis

D. Reiter syndrome

E. Scleroderma

A

A. Psoriatic arthritis

Hands are more commonly affected in psoriatic arthropathy, whereas feet are more commonly affected in Reiter’s

How well did you know this?
1
Not at all
2
3
4
5
Perfectly
9
Q
  1. A patient under joint care of cardiology and nephrology has bulky soft tissue nodules, well-marginated erosion, preserved joint spaces involving the wrists, elbows, shoulder and hip joints. Which is the most likely diagnosis?

A. Multicentric reticulohistiocytosis

B. Amyloidosis

C. Ochranosis

D. Wilson’s disease

E. Haemachromatosis

A

B. Amyloidosis

Multiple sites can be involved and there is characteristic preservation of joint spaces.

How well did you know this?
1
Not at all
2
3
4
5
Perfectly
10
Q

15) On plain radiographs of the hands in a middle-aged male patient complaining of bilateral joint pain and swelling, which single feature is most likely to support a diagnosis of psoriatic arthritis over rheumatoid arthritis?

a. new bone formation

b. joint space loss

c. periarticular osteoporosis

d. periarticular erosions

e. soft-tissue swelling

A

a. new bone formation

New bone formation is the hallmark finding of psoriatic arthritis and is not seen in rheumatoid arthritis. Conversely, periarticular osteoporosis is seen in rheumatoid but is not a feature of psoriatic arthritis.

Both conditions may cause soft-tissue swelling (typically a sausage digit in psoriatic arthritis), joint space loss and erosions, which are marginal in psoriatic and marginal and/or central in rheumatoid arthritis.

Another distinguishing factor is the distribution of involved joints in the hands, which is typically, but not always, interphalangeal in psoriatic and metacarpophalangeal in rheumatoid arthritis.

How well did you know this?
1
Not at all
2
3
4
5
Perfectly
11
Q

22) A 32-year-old woman with a long history of right knee pain undergoes radiography for atraumatic swelling of the joint and is found to have an effusion and soft-tissue swelling but no other findings. MRI shows a large anterolateral lobular intra-articular mass of low signal on T1W and T2W images, and a blooming artefact is seen on gradient echo sequences. What is the most likely condition?

a. malignant fibrous histiocytoma

b. pigmented villonodular synovitis

c. synovial osteochondromatosis

d. Baker’s cyst

e. intra-articular haematoma

A

b. pigmented villonodular synovitis

Pigmented villonodular synovitis is a benign proliferative disorder of the synovium that has a propensity for young to middle-aged adults and typically has a long history.

On plain radiographs, joint space and bone mineralization are typically preserved until late in the disease, but softtissue swelling or effusion may be apparent early on.

Haemorrhage is relatively common and can result in haemarthrosis and blooming artefact seen on gradient echo MRI sequences.

Malignant fibrous histiocytoma is the most common soft-tissue sarcoma after age 50.

Synovial osteochondromatosis is more common in men and is characterized by proliferation of the synovium with formation of cartilaginous nodules (that often calcify), but does not show haemorrhage.

Baker’s cyst has synovial fluid characteristics on MRI and is located posterior to the joint.

How well did you know this?
1
Not at all
2
3
4
5
Perfectly
12
Q

(CNS) 32) A young man complains of early morning back pain and stiffness, and undergoes plain radiographs followed by MRI of the whole spine. Which single feature is most likely to suggest a diagnosis of psoriatic arthritis over ankylosing spondylitis?

a. syndesmophytes

b. parasyndesmophytes

c. asymmetrical sacroiliitis

d. ankylosis

e. patchy bone marrow oedema

A

b. parasyndesmophytes

Seronegative spondyloarthritis is an umbrella term for inflammatory joint or spinal conditions that are not associated with rheumatoid factor or rheumatoid nodules.

There are five described subgroups: ankylosing spondylitis, psoriatic arthritis, arthritis associated with inflammatory bowel disease, reactive arthritis (e.g. Reiter’s syndrome) and an undifferentiated subgroup.

The subgroups may overlap both clinically and radiologically, and the diagnosis is more easily made on the basis of clinical history and examination.

Imaging plays a limited role in differentiation, particularly early in the disease when there can be considerable overlap of appearances.

The main exception is the identification of parasyndesmophytes, which are seen almost exclusively in psoriatic arthropathy.

In addition, bone marrow oedema can involve the entire vertebral body in psoriatic arthritis, which may be a further useful distinguishing feature.

Undifferentiated spondyloarthritis is diagnosed when there is no clinical or radiological evidence of sacroiliitis.

All types may eventually progress to ankylosis.

How well did you know this?
1
Not at all
2
3
4
5
Perfectly
13
Q

40) On plain radiographs, which of the following is the most specific indicator of prosthetic loosening following total hip replacement?

a. sclerosis at the tip of the femoral component

b. 3 mm, lucent line at the cement/prosthesis interface

c. heterotopic bone formation

d. periprosthetic fracture

e. femoral periosteal reaction

A

b. 3 mm, lucent line at the cement/prosthesis interface

The artefact created by metallic prostheses on CT and MRI means that plain radiography has an important role in the evaluation of postoperative arthroplasty joints.

Cemented prostheses may normally show a 1–2 mm lucent line at cement interfaces, but definite loosening is diagnosed with progressive widening of this zone.

Other specific indicators of loosening in both cemented and uncemented prostheses include migration of components or a new abnormality of alignment.

Periosteal reaction and sclerosis can be normal findings, particularly in uncemented prostheses.

Serial imaging is often required to confirm the diagnosis of loosening.

How well did you know this?
1
Not at all
2
3
4
5
Perfectly
14
Q

46) On plain radiographs of the hands, hyperflexion of the proximal interphalangeal joint of the index finger, with hyperextension of the distal interphalangeal joint of the same finger, describes which deformity?

a. swan-neck

b. Boutonnie`re

c. mallet finger

d. baseball finger

e. Z-deformity

A

b. Boutonniere

The Boutonniere deformity is commonly caused by injury or inflammatory conditions such as rheumatoid arthritis, and more commonly affects the index than middle fingers.

It consists of four stages. Stages 1 and 2 are mild and moderate, passively correctable extension lag, whereas stages 3 and 4 are mild and advanced flexion contractures.

The proximal flexion deformity is due to disruption of the central slip of the extensor tendon, with the proximal phalanx herniating through the defect and the lateral slips lying on either side. The position of the proximal phalanx stretches the lateral slips and pulls the distal phalanx into extension.

Swan-neck deformity has similar causes but the opposite configuration, with extension at the proximal interphalangeal joint and flexion distally.

Mallet (or baseball) fingers have a passively correctable flexion deformity of the distal interphalangeal joint caused by avulsion of the extensor digitorum tendon by a hyperflexion injury.

Z-deformity is the name given to a Boutonniere-type deformity seen in the thumb.

How well did you know this?
1
Not at all
2
3
4
5
Perfectly
15
Q

58) An elderly female patient has plain radiographs performed in an outpatient clinic for bilateral painful, stiff hips, which demonstrate joint space narrowing. Which additional feature is more likely to support a diagnosis of rheumatoid arthritis rather than osteoarthritis?

a. eccentric joint space loss

b. soft-tissue swelling

c. subchondral sclerosis

d. subchondral cysts

e. protrusio acetabuli

A

e. protrusio acetabuli

Even or eccentric joint space reduction representing cartilage loss is seen in both types of arthritis and not a distinguishing diagnostic feature.

Although osteoarthritis is said to be classically eccentric, this is difficult to assess accurately on many hip radiographs, as they are not routinely taken in the upright, weight-bearing position.

Subchondral sclerosis and cysts are typically associated with degenerative osteoarthritis.

Although soft-tissue swelling is a feature of rheumatoid arthritis, the depth of the hip joint and the copious surrounding soft tissues mean that any synovial swelling is unlikely to be appreciated clinically or radiologically.

Subtle osteophytes (in osteoarthritis) or erosive change/osteoporosis (in rheumatoid arthritis) can distinguish between the two entities.

An inflammatory cause should be considered in young adults with hip pain and, if protrusio or other abnormalities are found, the sacroiliac joints should be examined.

How well did you know this?
1
Not at all
2
3
4
5
Perfectly
16
Q

65) A 40-year-old woman presents to the emergency department with a painful, stiff shoulder, 12 hours after undergoing arthrography of the same joint. She describes onset of symptoms 8 hours previously with progressive worsening. She feels otherwise well with a temperature of 37.3 There is no overt joint swelling or overlying erythema. What is cause?

a. septic arthritis

b. chemical synovitis

c. joint haemarthrosis

d. joint effusion

e. allergic contrast reaction

A

b. chemical synovitis

Post-arthrography pain due to sterile chemical synovitis is the most common complication of the procedure, typically beginning after 4 hours and peaking at 12 hours. Other, less common, immediate and short-term complications include allergic contrast reaction (rare in intra-articular injections), introduction of infection and vasovagal reaction.

How well did you know this?
1
Not at all
2
3
4
5
Perfectly
17
Q

73) A 70-year-old man has plain radiographs of the hands and knees for joint pain and swelling, which show joint space narrowing and chondrocalcinosis. Which additional finding would support a diagnosis of haemochromatosis over pseudogout?

a. periarticular calcium deposition

b. metacarpal hooked osteophytes

c. eccentric joint space narrowing

d. large subchondral cysts

e. intra-articular loose bodies

A

b. metacarpal hooked osteophytes

Haemochromatosis is excess iron deposition in the tissues and can be either primary (autosomal recessive genetic disorder) or secondary to ineffective erythropoiesis or iron overload.

Skeletal manifestations include osteoporosis (which is proportional to the extent of iron deposition), small subchondral cysts, arthropathy (50%), concentric joint space narrowing, chondrocalcinosis, and characteristic hooked osteophytes on the radial aspect of the metacarpal heads.

Other organs can be affected by iron deposition, most commonly the brain, liver, pancreas and spleen.

How well did you know this?
1
Not at all
2
3
4
5
Perfectly
18
Q

80) A 72-year-old women presents to the rheumatologist with a long history of shoulder pain affecting her dominant arm that began at night with associated stiffness, but has suddenly worsened over the past few weeks. Radiographs show a superiorly subluxed humeral head forming a pseudarthrosis with the acromion, glenohumeral joint space loss, humeral head collapse with cysts and sclerosis, and periarticular soft-tissue calcification. Ultrasound scan demonstrates an effusion with widespread degeneration of the rotator cuff and a complete tear of the supraspinatus tendon. Examination of aspirated joint fluid shows calcium hydroxyapatite crystals. What is the most likely diagnosis?

a. Milwaukee shoulder

b. pseudogout

c. myositis ossificans progressiva

d. erosive osteoarthritis

e. scleroderma

A

a. Milwaukee shoulder

Milwaukee shoulder is a crystal deposition disease of basic calcium phosphate, predominantly affecting elderly women and resulting in a dysfunctional shoulder from destruction of the rotator cuff. It is often bilateral but always involves the dominant side.

Radiographic findings include superior subluxation of the humerus due to loss of the superior rotator cuff, often forming a pseudarthrosis with the clavicle or acromion.

Glenohumeral degeneration manifests as sclerosis and collapse of the humeral head, joint space narrowing and osteophyte formation.

Erosion at the site of rotator cuff insertion and periarticular soft-tissue calcification is also a feature.

Examination of effusion fluid is stereotypical, revealing spheroid-shaped aggregates of hydroxyapatite crystals.

The condition is also seen in the knee, where, unlike osteoarthritis, it predominates in the lateral compartment.

How well did you know this?
1
Not at all
2
3
4
5
Perfectly
19
Q

90) A 30-year-old man visits his general practitioner complaining of recent onset of acne and discharging pustules on his palms. He has a history of several years of pain and swelling at the medial end of his right clavicle. Radiographs of the shoulder demonstrate hyperostosis and early ankylosis of the sternoclavicular joint. What is the most likely diagnosis?

a. SAPHO syndrome

b. suppurative osteomyelitis

c. psoriatic arthritis

d. Reiter’s disease

e. recurrent multifocal osteomyelitis

A

a. SAPHO syndrome

Synovitis, acne, pustulosis, hyperostosis and osteitis (SAPHO) syndrome is a term encompassing several disease entities that demonstrate an association between rheumatological and cutaneous lesions. There may be a delay of several years between the onset of osseous symptoms and cutaneous manifestations. It is thought to be similar to chronic recurrent multifocal osteomyelitis in children. The dominant radiographic abnormality is new and bizarre bone proliferation, with the sternoclavicular joint affected in 70–90% of cases.

How well did you know this?
1
Not at all
2
3
4
5
Perfectly
20
Q

95) A 70-year-old man attends a 6-week follow-up appointment after cemented total hip arthroplasty, complaining of a poor range of motion. Radiographs taken during the appointment show small areas of pericapsular bone, and formation of small bony spurs at the acetabular margin. CT demonstrates these areas to have well-defined mineralization peripherally and indistinct centres. Which of the following processes are responsible?

a. femoral component loosening

b. heterotopic ossification

c. periprosthetic fracture

d. postoperative infection

e. stress shielding

A

b. heterotopic ossification

Heterotopic ossification, also known as myositis ossificans, is a benign, self-limiting process of ossification occurring within skeletal muscle.

Seventy-five per cent of cases are due to trauma (including iatrogenic trauma), with other causes including paralysis, burns, tetanus and intramuscular haematoma.

The areas of new bone are surrounded by fibrotic connective tissue, which can be seen as a soft-tissue mass on MRI.

Some heterotopic ossification is seen in half of all total hip replacements, with one-third considered clinically significant. It is classified radiographically according to the Brooker classification.

How well did you know this?
1
Not at all
2
3
4
5
Perfectly
21
Q

(CVS) 94) A 56-year-old female has arthralgia. A chest radiograph shows erosion of the lateral ends of the clavicles, superior notching of the third to fifth ribs, and narrowing of the humeral–acromial space. What is the most likely diagnosis?

a. hyperparathyroidism

b. rheumatoid arthritis

c. osteoarthritis

d. cleidocranial dysostosis

e. neurofibromatosis

A

b. rheumatoid arthritis

Erosion of the lateral ends of the clavicle is seen in several conditions, including rheumatoid arthritis, hyperparathyroidism and cleidocranial dysostosis. The associated rib notching and loss of the space between the humeral head and acromion (due to wasting/rupture of supraspinatus) are typically seen with rheumatoid arthritis. Clavicle erosions are not features of neurofibromatosis or osteoarthritis.

How well did you know this?
1
Not at all
2
3
4
5
Perfectly
22
Q

(GU) 98) A plain abdominal radiograph is acquired for left-sided abdominal pain. The lumbar spine is osteoporotic with intervertebral disc space narrowing, vacuum phenomenon and calcification. Marginal osteophytes and endplate sclerosis are also present. In addition to nephrocalcinosis, a radio-opaque calculus is noted along the path of the left ureter. Which of the following is the most likely pattern of inheritance?

a. autosomal recessive

b. autosomal dominant

c. autosomal dominant with partial penetrance

d. mitochondrial

e. X-linked

A

a. autosomal recessive

Alkaptonuria is the unifying diagnosis and is usually autosomal recessive. In this condition the absence of homogentisic acid oxidase causes accumulation of homogentisic acid, which is excreted in urine and sweat. Ochronotic deposition in the cardiovascular system causes atherosclerosis, aortic and mitral valve calcification, and myocardial infarction.

How well did you know this?
1
Not at all
2
3
4
5
Perfectly
23
Q

15 A lady suffers from long-standing rheumatoid arthritis. She is noted to have splenomegaly on a clinic visit. A diagnosis of Felty’s syndrome is suspected clinically. Which of the following is not a recognised feature of Felty’s syndrome?

(a) Ulceration

(b) Neutropaenia

(c) Skin pigmentation’
·
(d) Weight loss

(e) Low titres of rheumatoid factor

A

(e) Low titres of rheumatoid factor

Felty’s syndrome is characterised by the combination of longstanding rheumatoid arthritis, splenomegaly and neutropaenia. Associated features include weight loss, leg ulceration, and brown skin pigmentation. Almost all have high titres of rheumatoid factor.

How well did you know this?
1
Not at all
2
3
4
5
Perfectly
24
Q

16 A patient presents with back pain. He is found to be HLA- 827 positive and a diagnosis of ankylosing spondylitis is suspected. Plain films of the spine are requested. Which of the following would be the least supportive of this diagnosis?

(a) Calcification of the anterior longitudinal ligament

(b) Osteitis

(c) Syndesmophytes

(d) Sclerosis of the costotransverse joints

e) Ankylosis of the costovertebral joints

A

(a) Calcification of the anterior longitudinal ligament

Ossification of the posterior, rather than anterior longitudinal ligament is typical of ankylosing spondylitis.

How well did you know this?
1
Not at all
2
3
4
5
Perfectly
25
Q

(Ped) 31 Which of the following features is not a feature of Juvenile Chronic Arthritis?

(a) Growth retardation

(b) Iridocyclitis

(c) Rheumatoid factor positive

(d) Spinal involvement typically precedes peripheral arthritis

(e) Oligoarthritis

A

(c) Rheumatoid factor positive

JCA by definition sjarts < 16 years old, it can be oligo- or poly- arthritis and the majority of cases are seronegative. The large joints are typically affected first, diffuse ankylosis is common and periosteal reaction is typically present. Although there is often overgrowth of the epiphyses due to increased perfusion, there is usually growth retardation due to early closure of the growth plates.

How well did you know this?
1
Not at all
2
3
4
5
Perfectly
26
Q

41 A patient with joint pain has plain radiographs of the hands and spine. The differential diagnosis is considered to be between psoriatic arthropathy and rheumatoid arthritis. Which of the following features is more common in rheumatoid arthritis rather than psoriatic arthropathy?

(a) Phalangeal enthesophytes

(b) Involvement of the distal interphalangeal joints

(c) Fusiform soft tissue swelling of the digits

(d) Asymmetric joint involvement

(e) Involvement of the wrist joints

A

(e) Involvement of the wrist joints

The wrists are more commonly involved in rheumatoid arthritis

How well did you know this?
1
Not at all
2
3
4
5
Perfectly
27
Q

49 The pelvic radiograph of a young woman reveals bilateral, symmetrical triangular areas of subchrondral sclerosis on the inferior aspect of the iliac side of the joint. The joint spaces are normal and the joint margin is well-defined. There is no evidence of bone or joint disease elsewhere. What is the most likely diagnosis?

(a) Osteitis condensans ilii

(b) Infection

(c) Osteoarthritis

(d) Hyperparathyroidism

(e) Alkaptonuria

A

(a) Osteitis condensans ilii

Osteitis condensans ilii produces this radiographic appearance, typically in young frlultiparous women. Possible differential diagnoses include: ankylosing spondylitis, rheumatoid arthritis (both would be associated· with joint space changes) and Paget’s disease.

How well did you know this?
1
Not at all
2
3
4
5
Perfectly
28
Q

3 A patient presents with joint pain and plain radiographs are taken. Which of the following features would favour the diagnosis of osteoarthritis over rheumatoid arthritis?

(a) Marginal erosions

(b) ‘High-riding’ shoulder

(c) Periarticular osteoporosis

(d) Superolateral migration of the femoral head

(e) Involvement of the proximal joints of the hand

A

(d) Superolateral migration of the femoral head

In RA , the femoral head tends to migrate axially, whilst in osteoarthritis, it tends to migrate superolaterally.

How well did you know this?
1
Not at all
2
3
4
5
Perfectly
29
Q

10 A gentleman presents with knee swelling, pain and stiffness. Plain radiographs and subsequent MR imaging were performed, and a diagnosis of pigmented villonodular synovitis is suspected. Which of the following is not typical of this disease?

(a) Preservation of bone density

(b) Synovial low signal intensity on gradient echo MR imaging

(c) Well-defined erosions on both sides of the joint

(d) Joint space narrowing early in the disease

(e) Joint effusion

A

(d) Joint space narrowing early in the disease

PVNS is a proliferation of synovium that occurs in joints, bursae and tendon sheaths. The joint space is well preserved until late in the disease.

How well did you know this?
1
Not at all
2
3
4
5
Perfectly
30
Q

30 A pelvic radiograph reveals sacroilitis. This is most prominent on the lower and middle thirds of the joints, particularly on the iliac side. The changes are bilateral and symmetrical. These appearances are most commonly seen in which of the following conditions?

(a) Reiter’s syndrome

(b) Rheumatoid arthritis

(c) Gouty arthritis

(d) Osteoarthritis

(e) Ankylosing spondylitis

A

(e) Ankylosing spondylitis

Ankylosing spondylitis typically produces a bilateral, symmetrical sacroiliitis, whilst the other conditions typically result in bilateral but asymmetrical disease. Other conditions producing bilateral, ·symmetrical sacroiliitis include: inflammatory bowel disease, psoriatic arthropathy, osteitis condensans ilii, hyperparathyroidism and paraplegia.

How well did you know this?
1
Not at all
2
3
4
5
Perfectly
31
Q

@# 41 A 35 year old woman presents with gradually progressive knee pain over several years with locking of the joint. Plain radiographs show multiple round, well defined calcified loose bodies in the joint with no osteoporosis, and widening of the joint space. Which of the following is the likeliest diagnosis?

(a) Synovial osteochondromatosis

(b) Pigmented villonodular synovitis

(c) Lipoma arborescens

(d) Synovial sarcoma

(e) Osteochondritis dissecans

A

(a) Synovial osteochondromatosis

Cross sectional imaging shows a soft tissue mass of near water attenuation on CT containing calcifications, and a lobulated intraarticular mass which is isointense to muscle on T1 W and hyperintense on T2W with foci of low signal intensity.

How well did you know this?
1
Not at all
2
3
4
5
Perfectly
32
Q

45 A 25 year old man presents with chronic back pain. X-rays show bilateral irregularity of the sacro-iliac joints and paravertebral ossification on the AP view. Radiographs of the feet demonstrate destruction of the interphalangeal joint of the great toe with an exuberant periosteal reaction and bony proliferation at the distal phalangeal base. Which of the following is the likeliest diagnosis?

(a) Ankylosing spondylitis

(b) Rheumatoid arthritis

(c) Psoriatic arthritis

(d) Reiter’s syndrome

(e) Inflammatory bowel disease

A

(c) Psoriatic arthritis

The changes in the feet are characteristic of psoriatic arthropathy with soft tissue swelling and erosive changes leading to a ‘pencil-in-cup’ deformity. When seen, the changes at the interphalangeal joint of the great toe are pathognomonic. The SI joints are involved in 40% of cases and involvement can be either uni- or bi-lateral. As with Reiter’s syndrome, a large bulky paravertebral area of ossification (‘floating osteophyte’) is often seen.

How well did you know this?
1
Not at all
2
3
4
5
Perfectly
33
Q

56 A 50 year old man presents with gradual onset pain in multiple joints. Radiographs and MRI examinations show bilateral olecranon bursal effusions along with well defined, homogeneous, enhancing soft tissue masses which are isointense to muscle on T1 W and hypo intense on T2W which involve the wrist, ankle and knee joints. What is diagnosis?

(a) Rheumatoid arthritis

(b) Gout

(c) Pseudogout

(d) Psoriasis

(e) Amyloidosis

A

(b) Gout

Gout is polyarticular in 10% of cases. The soft tissue components have characteristic appearances and may calcify. Bilateral olecranan bursal effusions are considered pathognomonic

How well did you know this?
1
Not at all
2
3
4
5
Perfectly
34
Q

57 A 34 year old woman is found to have bilateral hilar lymphadenopathy on CXR along with elevated serum ACE levels. Which of the following radiological features would not be in keeping with the likely diagnosis?

(a) Acro-osteolysis of the distal phalanges

(b) Erosions involving the joints of the hand

(c) A lacelike trabecular pattern in the metaphysis of the metacarpals

(d) More advanced changes in the middle and distal phalanges than the proximal phalanges and the metacarpal bones

(e) Sclerosis of multiple vertebral bodies

A

(b) Erosions involving the joints of the hand

The joints are rarely involved in bone sarcoidosis. Other musculoskeletal features include the presence of paravertebral soft tissue masses, osteolytic lesions of the skull and well defined cystic bone lesions.

How well did you know this?
1
Not at all
2
3
4
5
Perfectly
35
Q

59 A 27 year old man presents with ongoing pain in the region of the sternum. There is no history of trauma. On examination there is tenderness, pain and swelling over the sternoclavicular joints; he is systemically well but is noted to have yellowish intradermal blisters on his hands. There is sclerosis and ankylosis of the right SCJ, with hyperostosis and hypertrophy of the left SCJ noted. What is the likely diagnosis?

(a) Ankylosing spondylitis

(b) Caffey’s disease

(c) Osteomyelitis

(d) Paget’s disease

(e) SAPHO syndrome

A

(e) SAPHO syndrome

SAPHO syndrome (synovitis, acne, pustulosis, hyperostosis and osteitis) is a rare inflammatory bone disorder that is associated with skin manifestations. The associated hyperostosis is frequently located at the points of the bone where tendons attach, and the SCJs are the commonest site affected (70-90% ). The patient is the wrong age for options (b) and ( d); osteomyelitis is the main differential, but the patient is systemically well and the other features support a diagnosis of SAPHO.

How well did you know this?
1
Not at all
2
3
4
5
Perfectly
36
Q

62 A 45 year old woman has hand radiographs for multiple joint pains. The radiographs show fusion of the distal interphalangeal joints. Which is the least likely diagnosis?

(a) Erosive osteoarthritis

(b) Reiter’s syndrome

(c) Rheumatoid arthritis

(d) Psoriatic arthropathy

(e) Ankylosing spondylitis

A

(c) Rheumatoid arthritis

In rheumatoid arthritis, the changes classically affect the MCP and PIP joints but spare the DIP joints.

How well did you know this?
1
Not at all
2
3
4
5
Perfectly
37
Q

67 A 60 year old man who had a THR a year previously presents with discomfort and a clicking sensation in that hip. Which of the following features are considered normal?

(a) 3 mm radiolucent zone at the cement bone interface

(b) Endosteal sclerosis at the tip of the femoral component

(c) 5 mm subsidence of the femoral component

(d) Progressive metal bead shedding

(e) Cement fracture

A

(c) 5 mm subsidence of the femoral component

Serial radiographs are required for full assessment of the joint replacement. As a minimum the British Orthopaedic Association suggest post-operative radiographs, then check radiographs at 1 year, 5 years and every subsequent 5 years. Other signs include progressive radiolucency at the cement bone interface, the presence of well-defined lucencies at the interface which may suggest granulomatous disease and acetabular migration. A degree of subsidence of the femoral component is considered normal and either progressive change or an absolute value of greater than 10 mm is abnormal.

How well did you know this?
1
Not at all
2
3
4
5
Perfectly
38
Q

(Ped) 70 A 6 year old boy presents with a 2 day history of limp and right hip/ anterior thigh pain. He has a low-grade fever and the ESR is 30 mm/hr. An US is requested to help differentiate transient synovitis (TS) from septic arthritis. Which of the following is true?

(a) Capsular distension of 2 mm diagnoses an effusion

(b) Definitive US features can exclude septic arthritis

(c) Synovial membrane thickening favours a diagnosis of TS

(d) US is more sensitive than hip X-Ray for detecting Perthe’s disease

(e) US is performed with the hip in internal rotation

A

(a) Capsular distension of 2 mm diagnoses an effusion

TS presents at 5-10 years and is the commonest cause of a nontraumatic limp in this age group. US is performed with the child supine and the hip in the neutral position, or in slight external rotation. US typically shows an effusion (capsular distension > 2 mm), without synovial thickening. It is a diagnosis of exclusion, radiographs may help differentiate Perthe’s disease; fluid aspiration helps exclude septicarthritis.

How well did you know this?
1
Not at all
2
3
4
5
Perfectly
39
Q
  1. A 35 year old man presents with increasing stiffness in his knee and soft-tissue swelling around the joint. Plain films show multiple areas of irregular cyst-like radiolucencies in the distal femur. There are no areas of abnormal calcification and there is no evidence of periarticular osteoporosis. MR shows a low signal joint effusion on both T1 and T2 sequences. The most likely diagnosis is:

a. Synovial osteochondromatosis

b. Pigmented villonodular synovitis

c. Osteoarthritis

d. Reiter’s syndrome

e. Osteomyelitis

A
  1. b. Pigmented villonodular synovitis

Pigmented villonodular synovitis is a relatively rare condition which usually presents in the third or fourth decade. It is a monoarticular, painful disease which causes a decreased range of movement at the affected joint. It is most common at the knee (80%) followed by the hip, ankle, shoulder and elbow. Haemorrhagic ‘chocolate’ effusion is characteristic. Low signal effusion on all sequences at MR is characteristic. There is no calcification or osteoporosis, and joint space narrowing is a late feature.

How well did you know this?
1
Not at all
2
3
4
5
Perfectly
40
Q
  1. In a 21 year old man with symptoms of chronic back pain, pain in his feet, particularly the great toe and metatarsophalangeal joints, and bilateral sacroiliitis on plain films, the most likely diagnosis is:

a. Ankylosing spondylitis

b. Gout

c. Inflammatory bowel disease-related arthropathy

d. Reiter’s syndrome

e. Psoriatic arthritis

A
  1. d. Reiter’s syndrome

Reiter’s syndrome is the association of urethritis, conjunctivitis and mucocutaneous lesions. Sacroiliitis is usually bilateral but often persists asymmetrically. There is an association with the HLA B27 antigen. Reiter’s has a predeliction for the great toe and metatarsophalangeal joints.

How well did you know this?
1
Not at all
2
3
4
5
Perfectly
41
Q

(CVS) 11. A 51 year old man with long standing history of an erosive arthropathy of the acromioclavicular joints and bilateral arthropathy in his hands subsequently develops progressive shortness of breath. The most likely abnormality on his chest radiograph would be:

a. Cavitating nodules

b. Peripheral basal reticulonodular shadowing

c. Cardiomegaly

d. Bronchiectasis

e. Pleural effusion

A
  1. e. Pleural effusion

Pleural effusion is the commonest thoracic manifestation of rheumatoid arthritis, much more common in men (M:F 9:1). It is unilateral in the vast majority of cases. The fluid is an exudate with low sugar content and is often seen in the absence of other pulmonary changes.

How well did you know this?
1
Not at all
2
3
4
5
Perfectly
42
Q
  1. A routine pre-operative chest X-ray in a 62 year old woman shows bilateral erosion of the distal clavicles. Which one of the following conditions might be responsible?

a. Hypoparathyroidism

b. Rheumatoid arthritis

c. Langerhans’ cell histiocytosis

d. Ankylosing spondylitis

e. Sarcoidosis

A
  1. b. Rheumatoid arthritis

Myeloma, hyperparathyroidism, metastases, cleidocranial dysplasia and Gorlin basal cell nevus syndrome all cause absence of the outer end of the clavicle. Destruction of the medial end of the clavicle is caused by metastases, infection, lymphoma, eosinophilic granuloma, rheumatoid arthritis and sarcoma.

How well did you know this?
1
Not at all
2
3
4
5
Perfectly
43
Q
  1. A 75 year old woman presents with increasing pain in her left hip. She had a total hip replacement eight years ago on this side which has been asymptomatic ever since. Plain radiographs demonstrate a lucent line at the bone cement interface of the femoral component. The likely cause for this is:

a. Infection

b. Metastasis

c. Loosening

d. Myeloma

e. Trauma

A
  1. c. Loosening

Early changes (less than six months) are almost always due to infection. Up to four years, infection remains the most likely cause, but after this point loosening becomes more common. Every year, 38 000 hips are replaced in the UK. A routine postoperative film is usually performed; an excessively varus stem may lead to loosening.

44
Q
  1. A 57 year old man with increasing pain and stiffness in his hands and feet and worsening back pain presents to his GP. Plain films of his hands show sclerosis of the terminal phalanges and several ‘pencil-in-cup’ erosions. There is destruction of the interphalangeal joint of his right great toe with exuberant periosteal reaction. There is also erosion of the posterior margin of the calcaneus. The most likely diagnosis is:

a. Reiter’s syndrome

b. Ankylosing spondylitis

c. Rheumatoid arthritis

d. Psoriatic arthritis

e. Osteoarthritis

A
  1. d. Psoriatic arthritis

This is usually HLA B27 positive and is associated with skin and nail changes in the majority of cases. The hands are often described as having sausage digits, and erosions with ill-defined margins are characteristic. Sacroiliitis is often present and most often bilateral. Within the axial skeleton, there is often large bulky vertically orientated soft-tissue ossification giving a ‘floating’ osteophyte appearance.

45
Q
  1. A dental radiograph of a 47 year old woman shows loss of the lamina dura of the majority of the teeth. Which of the following would be a possible cause?

a. Osteopetrosis

b. Hypoparathyroidism

c. Scleroderma

d. Sickle cell anaemia

e. Myeloma

A
  1. c. Scleroderma

The other causes of loss of the lamina dura include Cushing’s disease, Paget’s, hyperparathyroidism, osteoporosis, osteomalacia, leukaemia, metastases and Langerhans’ cell histiocytosis. Both osteopetrosis and hypoparathyroidism cause thickening of the lamina dura of the teeth.

46
Q

QUESTION 27
A 30-year-old women experiences, amongst other symptoms, recurrent episodes of painful swelling and stiffness of both hands. Antibodies against antinuclear antigens and double-stranded DNA are detected in peripheral blood samples. Radiographs of both hands are obtained during her assessment.
Which radiographic findings are most likely?

A Atrophic soft tissues, resorption of the terminal phalanges and soft tissue calcinosis

B Punched-out marginal erosions in an asymmetric distribution, with preservation of bone density

C Symmetrical abnormality of the MCPs of the index and middle finger, characterised by joint space narrowing, subarticular cysts and hook-like osteophytes

D Symmetrical soft tissue swelling, marginal erosions and juxta-articular osteopaenia

E Ulnar deviation at the MCPJs, without evidence of erosions

A

E Ulnar deviation at the MCPJs, without evidence of erosions

47
Q

@#e QUESTION 48
A 30-year-old man complains of intermittent painful swelling of his left knee over the past year. Radiographs show several small articular erosions, whilst subsequent MRI reveals foci of low T2/T2* signal intensity within the synovium. Which is the most likely diagnosis?

A Alkaptonuria

B Calcium pyrophosphate arthropathy

C Pigmented villonodular synovitis

D Psoriatic arthropathy

E Synovial chondromatosis

A

C Pigmented villonodular synovitis

48
Q

QUESTION 49
A 45-year-old man with hyperuricaemia is referred by the rheumarologists for an image-guided aspiration of a right ankle effusion. There is no previous history of note. A sample of the aspirate is sent for polarising light microscopy. What findings would confirm the clinical suspicion of gout?

A Rhomboid crystals

B Negatively birefringent needle shaped crystals

C Negatively birefringent rhomboid crystals

D Positively birefringent needle shaped crystals

E Positively birefringent rhomboid crystals

A

B Negatively birefringent needle shaped crystals

Monosodium urate crystals are needle-shaped and strongly negatively birefringent under polarising light. Weakly positively birefringent rhomboid crystals are characteristic of calcium pyrophosphate dihydate deposition.

49
Q

QUESTION 50
Which one of the following conditions is NOT a recognised component of the SAPHO spectrum?

A Arthritis

B Hyperostosis

c Osteomyelitis

D Pustulosis

E Synovitis

A

A Arthritis

50
Q

@#e The radiograph of a 40-year-old man with a painful knee shows multiple calcified loose bodies, each of similar size, within the joint. The joint space is preserved. What diagnosis is most likely?

A Calcium pyrophosphate arthropathy

B Gout

C Pigmented villonodular synovitis

D Rheumatoid arthritis

E Synovial osteochondromatosis

A

E Synovial osteochondromatosis

51
Q

QUESTION 65
A 50-year-old man presents to the Emergency Department following an injury to his right hand. No fracture is detected on plain radiographs, but the second and third MCPJs appear abnormal, with joint space narrowing, mbarticular cysts, hook-like osteophytes and flattening of the metacarpal heads. What is the most likely diagnosis?

A Alkaptonuria

B Gout

C Haemochromatosis

D Osteoarthritis

E Psoriatic arthritis

A

C Haemochromatosis

52
Q

A 34-year-old man with chronic back pain is referred by his GP for thoracic and lumbar spine radiographs. The GP is concerned about the possibility of ankylosing spondylitis. Which radiological feature is atypical for ankylosing spondylitis, and might suggest an alternative diagnosis?

A Ankylosis of the apophyseal joints

B Anterior longitudinal ligament calcification

C Osteophyte formation

D Sclerosis of the anterior corners of the vertebrae

E Vertebral body squaring

A

C Osteophyte formation

Syndesmophytes, rather than osteophytes, are characteristic features of ankylosing spondylitis. They are differentiated from osteophytes by their vertical orientation, as they represent ossification of the outer border of the annulus fibrosus. Progression and maturation of the syndesmophytes result in a ‘bamboo spine’.

53
Q

@#e QUESTION 68
A 50-year-old woman complains of painful swelling of the joints of the hands and wrists. Radiographs show evidence of an erosive arthropathy. Which radiological feature would favour a diagnosis of rheumatoid rather than psoriatic arthritis?

A Early reduction in bone mineralisation

B Erosions of the terminal tufts of the distal phalanges
C Joint ankylosis

D Pencil-in-cup deformities of the middle phalanges

E Periosteal reaction

A

A Early reduction in bone mineralisation

Juxta-articular osteopenia is one of the earliest radiographic abnormalities in rheumatoid arthritis, distinguishing it from psoriatic arthropathy, in which bone mineral density is preserved until late in the disease process.

54
Q
  1. A 28-year-old male presents with soft tissue swelling, pain, and reduction of motion in the small joints of his hands. Plain films of the hands show erosions at the metacarpophalangeal (MCP) joints and distal interphalangeal joints with periosteal reaction and enthesophytes. What is the most likely diagnosis?

A. Psoriatic arthropathy.

B. RA.

C. SLE.

D. Haemochromatosis.

E. Calcium pyrophosphate dihydrate crystal deposition disease.

A
  1. A. Psoriatic arthropathy.

Bone involvement before skin changes is evident in up to 20% of cases. Nail pitting or discolouration is common and correlated with the severity of the arthropathy. Five distinct manifestations have been described: oligoarthritis, polyarthritis (predominately distal IP joints), symmetric type (resembling RA), arthritis mutilans, and spondyloarthropathy. The characteristic distribution involves the small joints of the hands and feet, with or without spondyloarthropathy. Involvement in the hands tends to include distal IP as well as MCP or PIP joints, with early tuft resorption and distal IP erosive disease. The erosions become so severe that a ‘pencil-in-cup’ deformity and telescoping of the joint may occur. Bone density can be normal and the joint distribution is asymmetric. Similarly, sacroiliitis is asymmetric, unlike ankylosing spondylitis and syndesmophtes, which are non-marginal and asymmetric; in ankylosing spondylitis they are marginal and asymmetric. The spondyloarthropathy of psoriatric arthropathy is indistinguishable from that of reactive arthritis, clinical scenario (rash vs uveitis/urethritis) providing diagnosis. The arthropathies of CPPD disease and haemochromatosis are essentially identical radiographically. Chondrocalcinosis is commonly seen in the wrist (triangular fibrocartilage) and knee (menisci). The joints most affected are the knee, wrist and second and third MCPs of the hand: the IP joints tend to be spared. Early disease shows erosive change. More advanced disease demonstrates sclerosis, osteochondral fragments, and osteophytes. Subchondral cysts are common and large. RA is rarely found in the distal IP joints and periosteal reaction is not a feature. SLE is usually non-erosive and affects the MCP joints.

55
Q
  1. A 44-year-old female patient presents to the rheumatologists with a history of multiple painful joints for 2 years. She has synovitis clinically, confirmed on ultrasound, which involves the MCP joints bilaterally. PA and Norgaard views of the hands are requested and show small erosions in the distal radio-ulnar joint and the piso-triquetral joint, but no erosions at the MCP joints. There is widening of the scapholunate interval on the right side. There is anklyosis of the capitate to the hamate on the left. There is periarticular osteoporosis. Which of these features is atypical of RA?

A. Symmetrical disease.

B. Synovitis on ultrasound but no erosions radiographically.

C. Erosions noted in the radio-ulnar joint and radio-carpal joint preceding MCP erosions.

D. Bony ankylosis of the carpal bones.

E. Periarticular osteoporosis.

A
  1. D. Bony ankylosis of the carpal bones.

Whilst fibrous ankylosis of the carpal and tarsal bones does occur, bony ankylosis is extremely rare in RA. It is, however, common in JRA. There are a number of unusual findings, which if present should indicate a diagnosis other than RA. Productive bone change (e.g. periostitis or enthesopathy) is extremely unusual. Osteophytes are also uncommon in the absence of advanced associated osteoarthritic change. The exception to this is the distal ulna, a feature known as ulnar capping. The other features are all typical of RA.

56
Q
  1. You are carrying out an MRI on a patient with a known history of RA. The patient has minimal erosions on plain film, but severe arthralgia. She is being considered for biologic therapy. The clinicians have requested an MRI of her hands. This reveals symmetrical disease in both hands with areas of high signal on T2WI and low signal on T1WI around the triangular fibrocartilage complex (TFCC), the radio-carpal joint (RCJ), and the distal radio-ulnar joint (DRUJ). The abnormal areas at the TFCC and RCJ enhance following administration of gadolinium, the DRUJ does not. A delayed T1WI sequence displays uniform enhancement in all joints. What do these findings indicate?

A. Hypervascular pannus at the TFCC and RUJ, with fibrous pannus at DRUJ.

B. Fibrous pannus at the TFCC and RUJ, with joint effusion at DRUJ.

C. Hypervascular pannus at the TFCC and RUJ, with joint effusion at DRUJ.

D. Fibrous pannus at the TFCC and RUJ showing differential enhancement.

E. Fibrous pannus at the TFCC and RUJ with hypervascular pannus at DRUJ.

A
  1. C. Hypervascular pannus at the TFCC and RUJ, with joint effusion at DRUJ.

Hypervascular pannus is intermediate to high signal on T2WI and low signal on T1WI. It also enhances, retaining enhancement on delayed imaging. Joint effusions can be difficult to differentiate from hypervascular pannus on pre-contrast imaging. Following enhancement they show only delayed enhancement. Fibrous pannus is low signal on all sequences.

57
Q
  1. A 75-year-old man has a cemented right total hip replacement. On routine follow-up imaging he is noted to have a progressive well-delineated, rounded, focal area of lucency at the cement bone interface adjacent to the tip of the femoral stem. Which of the following given reasons is the most appropriate for this progressive lucency?

A. Aggressive granulomatous disease.

B. Primary loosening.

C. Cement fracture.

D. Normal finding.

E. Metal bead shedding.

A
  1. A. Aggressive granulomatous disease.

Well-delineated, rounded, focal areas of lucency at the cement bone interface, which are progressive, are suggestive of either infection or aggressive granulomatous disease. It can occur with both cemented and non-cemented components. Its origin is thought to be multifactorial. Metal, cement, or polyethylene fragments may penetrate the cement bone interface and induce a focal inflammatory foreign-body reaction, leading to osteolysis.
Primary loosening usually manifests as a wide (>2 mm) radiolucent zone at the cement–bone or metal–bone interface or a progressive radiolucent zone at the metal–cement interface. The radiolucent zones are not typically rounded. Cement fractures are thin linear lucent areas within the cement. They may be asymptomatic but are important to identify as they may lead to component failure. Metal bead shedding is defined as opaque microfragments separated from the porouscoated femoral stem. Metal beads can be seen on immediate postoperative radiographs, as a consequence of the stem insertion. Bead shedding might later occur with loose non-cemented components, reflecting micro-motion of the stem. These metal beads are seen in the soft tissue adjacent to the hip replacement and their increase in number on follow-up indicates loosening.

58
Q
  1. An 18-year-old male patient presents to the rheumatologists with a history of proximal right tibial pain and sternal pain. The patient has a history of psoriasis and is also being seen by the dermatologists with palmoplantar pustulosis. Plain films of the sternum indicate sclerosis of the manubrium and erosive disease in the sternoclavicular joint. Plain films of the tibia show a lucent lesion in the proximal tibial metaphysis with associated periosteal reaction. An MRI shows high signal on STIR in the proximal tibial metaphysis, with a cortical defect. This area enhances on the T1 post gadolinium images, as does the periosteal region. A bone biopsy of the region is negative except for inflammatory cells. What is the most diagnosis?

A. Psoriatic arthropathy.

B. Synovitis, acne, pustulosis, hyperostosis, and osteitis (SAPHO) syndrome.

C. Chronic recurrent multifocal osteomyelitis (CRMO).

D. Chronic osteomyelitis.

E. Aseptic necrosis

A
  1. B. SAPHO syndrome.

The most common presentation for this condition is acne and palmoplantarpustulosis with changes in the sternocostoclavicular region where hyperostosis is seen, often with some erosive change. The condition can also cause a number of manifestations in the spine, ranging from focal hyperostosis of one or more lumbar vertebrae, to a syndrome strikingly similar to psoriatic spondylarthropathy. In the appendicular skeleton hyperostosis is commonly seen. SAPHO can also give rise to manifestations identical to osteomyelitis, but with no causative organism. In this respect there is overlap with CRMO syndrome. Patients are frequently human leucocyte antigen (HLA) B27 positive and psoriasis frequently coexists, leading some authors to suggest that SAPHO is a variant of psoriatic arthropathy, although this is not widely accepted.

59
Q
  1. You are reporting an MRI knee on a patient with moderately severe osteoarthritis (OA), as diagnosed on plain film radiography. The patient describes significant knee pain. Which of the following statements best describes the relationship between symptoms, plain film findings, and MRI findings?

A. The MRI findings correlate well with the severity of findings on plain film radiography.

B. MRI findings correlate well with the patient’s symptoms.

C. Plain film findings correlate well with the patient’s symptoms, unlike MRI.

D. Plain film and MRI both correlate well with the severity of the patient’s symptoms.

E. Symptoms, plain film findings, and MRI findings do not have a significant association with each other.

A
  1. A. The MRI findings correlate well with the severity of findings on plain film radiography.

MRI has been shown to correlate well with the severity of OA as depicted on plain film radiography. Neither MRI nor plain film appearances are significantly associated with the patient’s symptoms.

60
Q
  1. You are discussing OA with a rheumatologist. He/she is curious to know what radiological findings seen in early disease are associated with progressive, as opposed to stable, arthritis. All of these are associated with OA, but which is least likely to indicate progressive disease?

A. Increased uptake on isotope bone scan.

B. Grade 1 osteophytosis in the knee.

C. Osteochondral defect.

D. A focal area of high signal on T2WI and STIR in the subchondral bone.

E. A serpiginous subchondral line that is low signal on T2WI and T1WI, with an adjacent T2WI high signal line.

A
  1. B. Grade 1 osteophytosis in the knee.

While the Kellegren and Lawrence grading of OA is the most widely used scale for grading OA on plain films, early osteophyte formation is not definitively ‘arthritic’ change. Studies have shown that patients with this type of early change infrequently progress to developing more severe disease. Recent research in OA is focused on the impact of the subchondral bone on the disease, rather than hyaline cartilage. This is evidenced by increased uptake on bone scan being closely linked to progressive OA, even in patients with relatively normal joints on plain film. The foci of high signal on T2WI and STIR are bone marrow lesions that if persistent can indicate pathology in the subchondral bone, which can lead to arthritic change. The serpiginous line describes the classical finding of avascular necrosis. Both this and osteochondral defects lead to progressive OA change.

61
Q
  1. A 55-year-old female with a history of pulmonary sarcoidosis presents with pain in both hands. Which of the following findings on plain radiograph of the hands is atypical of skeletal sarcoidosis?

A. Cyst-like radiolucencies.

B. Joint space narrowing.

C. Bone erosions.

D. Subcutaneous soft tissue nodules/mass.

E. Lace-like pattern of bone destruction.

A
  1. B. Joint space narrowing.

Around 5–10% of patients with sarcoidosis demonstrate skeletal involvement. The phalanges in the hands and feet are most commonly affected. Joint space narrowing is unusual in sarcoidosis, unless neuropathic changes develop. Typical radiographic changes include cyst-like radiolucencies, a ‘lace-like’ pattern of bone destruction, bone erosions, and SC soft-tissue mass. These changes occurring in combination are diagnostic of sarcoidosis.

62
Q
  1. A 55-year-old female presents to the rheumatologists with a history of episodic swollen red joints over the previous 2 years. She also complains of left hip pain. The patient’s rheumatoid factor is not known at the time of requesting the radiographs. There is no other past medical history. The rheumatologists have requested bilateral hand and pelvis x-rays. The hand x-rays show bilateral asymmetric disease affecting the distal and proximal interphalangeal (IP) joints. In the affected distal IP joints there are central erosions, adjacent sclerosis, and marginal osteophytes. The first carpometacarpal joint in the left hand shows loss of joint space with osteophyte formation. The scaphoid-trapezium joint in the right hand also shows loss of joint space and adjacent sclerosis. In the left hip there is nonuniform loss of joint space, with associated subchondral cyst formation. What is the main differential?

A. RA.

B. Psoriatic arthritis.

C. Erosive OA.

D. Calcium pyrophosphate deposition (CPPD) arthropathy.

E. Ankylosing spondylitis.

A
  1. C. Erosive OA.

If the presence of erosions is ignored the pattern of disease is typical of OA. Erosive, or inflammatory, OA shows typical OA distribution, but with central erosions in the affected joint spaces in the distal IP, and less commonly proximal IP, joints. These central erosions in combination with marginal osteophytes give the classical gull-wing appearance to affected joints. Whilst individual joint appearance can be identical to psoriatic arthropathy, which can precede the development of the skin disease, the overall pattern is atypical, making it less likely. Also, while the distribution of CPPD arthropathy is often identical to OA, it would not cause erosions, and would usually be associated with chondrocalcinosis.

63
Q
  1. A patient presents to their GP with a complex history of acute episodes of severe tender inflamed joints, in particular around the knee. At present the patient has joint stiffness which is most pronounced in the evenings and mild joint pain. The patient has a past medical history of hypothyroidism. A plain film is requested which shows chondrocalcinosis and moderate degenerative change in the lateral tibiofemoral compartment and the patellofemoral compartment. Regarding CPPD disease, which of the following statements is the most appropriate?

A. The presence of chondrocalcinosis indicates a radiological diagnosis of pseudogout.

B. Pseudogout syndrome is the most common means of presentation for this disease.

C. Disproportionate involvement of the patellofemoral joint is the most frequently seen radiographic finding.

D. The presence of crystals displaying positive birefringence at polarized light microscopy allows for the definitive diagnosis of pyrophosphate arthropathy.

E. The presence of hypothyroidism is associated with the diagnosis.

A
  1. E. The presence of hypothyroidism is associated with the diagnosis.

Disorders associated with CPPD are the four Hs: hyperparathyroidism, haemochromatosis, hypothyroidism, and hypomagnesaemia. Use of nomenclature in this disorder is confused. Pseudogout is the clinical presentation of an acutely infl amed joint due to calcium pyrophosphate crystal deposition, and as such is not a radiological diagnosis. Pseudogout syndrome is the dominant feature in only 10–20% of cases of CPPD. Another 10–20% of cases are asymptomatic, whilst most present with symptoms identical to OA, with occasional fl ares. Disproportionate involvement of the patellofemoral joint is a characteristic feature, but is only occasionally seen. Pyrophosphate arthropathy is a description of the pattern of disease present due to crystal deposition, and as such is not a diagnosis made by analysis of joint aspirate.

64
Q
  1. A 30-year-old man presents with backache and morning stiffness. Examination reveals loss of spinal movement, uveitis, and upper zone end inspiratory fine crepitations on auscultation. Which of the following statements is most correct in relation to the radiological features of the underlying condition?

A. Romanus lesions (anterior or posterior spondylitis) are a late feature.

B. Syndesmophytes are better depicted on MRI than plain film.

C. Ankylosis involves the vertebral edges or centre.

D. Sacroiliac joint widening is not a feature.

E. Enthesitis appears as low signal within the ligaments on STIR imaging.

A
  1. C. Ankylosis involves the vertebral edges or centre.

The question refers to ankylosing spondyitis. Ankylosis involves the vertebral edges or centre, with bony extension through the disc. The former is thought to be secondary to a Romanus lesion, the latter an Andersson lesion. Romanus lesions are irregularities and erosions involving the anterior and posterior edges of the vertebral endplates and are the earliest changes of spondylitis depicted on conventional radiographs. On MRI an Andersson lesion is depicted as disc-related signal-intensity abnormalities of one or both vertebral halves of a discovertebral unit. They are often hemispherically shaped. MRI is better than conventional radiography at depicting Romanus lesions, Andersson lesions (spondylodiscitis), and most other abnormalities, although ankylosis is equally well detected by both modalities.
Syndesmophtyes are difficult to detect on MRI. Plain radiography is superior in this respect because of its superior spatial resolution; syndesmophtyes are seen as bony outgrowths of the anterior vertebral edges. They occur in 15% of the vertebrae of patients. Apical pulmonary fibrosis affects 1% of patients. Sacroiliac (SI) joint erosion and widening is an early feature, and this may initially be more prominent on the iliac side of the joint, as the cartilage on that side is normally thinner. Later in the disease, sclerosis and ankylosis occur and the SI joints become symmetrically fused. Enthesitis is most prominently seen when the interspinal ligaments, those that extend between the spinous processes, and the supraspinal ligaments are affected. Ligamentous involvement is characterized by an increased signal intensity on either STIR images or contrast-enhanced T1WI fat saturated sequences. It may be associated with osteitis of adjacent bone marrow in the spinous processes. Arthritis of the synovial joints (e.g. facet joints) and insufficiency fractures (often spontaneous or after minor trauma) are also features of the seronegative spondylarthritides.

65
Q
  1. A 47-year-old man presents with a progressive history of pain, swelling, and reduced range of movement affecting his right knee. Symptoms have been ongoing for 2–3 years. Locking is noted on examination. Radiography of his knee reveals multiple intra-articular calcifications. A supra-patellar joint effusion is also present. The joint space is maintained. What is the most likely diagnosis?

A. Neuropathic arthropathy.

B. Osteochondritis dissecans.

C. Osteochondral fracture.

D. PVNS.

E. Synovial osteochondromatosis.

A
  1. E. Synovial osteochondromatosis.

The primary form of this represents an uncommon benign neoplastic process with hyaline cartilage nodules in the subsynovial tissue of a joint, tendon sheath, or bursa. Secondary synovial chondromatosis is associated with joint abnormalities, such as mechanical or arthritic conditions, that cause intraarticular chondral bodies. The primary form of the disease predominantly affects men in the third to fifth decades. The knee is the most common site, but it is also seen in the hip, shoulder, elbow, and ankle; less commonly the MCP, IP, distal radioulnar, and acromioclavicular joints are involved. It can rarely involve extra-articular sites, the synovium about the tendons, or bursa. Clinical symptoms typically include pain, swelling, and restriction of the range of motion of the joint. Radiologic findings are frequently pathognomonic. Radiographs reveal multiple intraarticular calcifications (70–95% of cases) of similar size and shape distributed throughout the joint, with typical ‘ring-and-arc’ chondroid mineralization. Extrinsic erosion of bone is seen in 20–50% of cases. Juxtaarticular osteopenia is not typically apparent in synovial chondromatosis unless it is the result of disuse. CT is often diagnostic if the bodies are adequately mineralized and is particularly helpful for identifying characteristic ring-and-arc or punctate mineralization and the multiplicity of nodules in cases for which radiographic findings are normal or equivocal. Lack of mineralization of the bodies does occur in which cases MRI is very helpful to distinguish from, for example, PVNS or amyloid. The most common pattern on MRI (77% of cases) reveals low to intermediate signal intensity with T1WI and very high signal intensity with T2WI with hypointense calcifications.

66
Q
  1. A 24-year-old male patient is referred from the rheumatologists with a history of back pain and hip pain. Plain films are carried out. These show bilateral sacroiliitis with erosive change on the iliac side on the left, but sacral and iliac erosions on the right. The imaging of the spine reveals large non-marginal syndesmophytes in the thoracolumbar spine with a relatively normal lower lumbar spine. The patient also complains of foot pain and plain films reveal evidence of a retrocalcaneal bursitis with erosion of the calcaneus. Hand x-rays reveal small erosions asymmetrically in the distal IP joints in both hands. What is the most likely diagnosis?

A. Ankylosing spondylitis.

B. Reactive arthritis.

C. Psoriatic arthritis.

D. Erosive OA.

E. Adult Stills disease.

A
  1. C. Psoriatic arthritis.

Ankylosing spondylitis causes a symmetrical sacroiliitis. The syndesmophytes associated with this are marginal and fine. It also typically progresses superiorly from the lumbar spine. Both reactive arthritis and psoriatic arthritis cause an asymmetric sacroiliitis and the syndesmophytes are usually centred on the thoracolumbar spine and are non-marginal and bulky. Retrocalcaneal bursitis and erosions, whilst more common in reactive arthritis, can occur in psoriatic arthritis, and reactive arthritis would uncommonly affect the hands. Also, with all other factors being equal, psoriatic arthritis is much more common than reactive arthritis, even without the skin manifestations, which are absent in up to 20% at presentation.

67
Q
  1. A 45-year-old female is being investigated. She has a history of connective tissue disease. You are reviewing her imaging and trying to decide which connective tissue disease she has. Her hand x-rays reveal distal tuft resorption with cutaneous calcification. She also has erosion of the distal IP joints in the hands and the first carpometacarpal (CMC) joint. She has ulnar deviation deformity to the MCP joints in both hands on the Norgaard views, which corrects on the antero-posterior (AP) views. She also has an MRI scan of the pelvis which shows uniform high T2WI signal in the gluteal muscles bilaterally. From the list of connective tissues diseases below, select the one paired with a feature that is atypical for the disease but present in this patient?

A. Systemic sclerosis and high signal changes in muscles

B. SLE and deforming arthropathy

C. Systemic sclerosis and acro-osteolysis

D. Polymyositis and erosive arthropathy

E. Polymyositis and soft tissue calcification

A
  1. D. Polymyositis and erosive arthropathy.

There is a lot of overlap in the features of connective tissue diseases, with a number of patients labelled as mixed-connective tissue disease due to this. There are features of these conditions that can help differentiate them. SLE is a great mimic and can manifest in a myriad of ways. Erosive disease is not typically seen in SLE, but SLE does classically give a reducible deforming arthropathy of the hands, which is most pronounced on Norgaard views, but can appear completely normal on PA hand views. Avascular necrosis (AVN) and deforming arthropathy are not typically seen in systemic sclerosis. Acro-osteolysis and soft-tissue calcifications, especially in the fingertip pulps, are classical features of this disease. Polymyositis classically gives high T2WI signal in muscles due to myositis. It does not typically give erosions.

68
Q
  1. A 57-year-old female patient with a history of multiple myeloma is referred for imaging due to a history of arthralgia primarily affecting the hands. The patient describes early morning stiffness that eases through the day. The clinicians report a finding of synovitis clinically. Blood results have revealed a raised ESR. Hand x-rays are carried out which reveal sharply defined intra-articular marginal erosions at the MCP joints of the index and middle fingers bilaterally. The joint spaces are well preserved. There are also well-marginated subchondral cysts noted in the carpal bones, again with joint space preservation. Soft tissue nodules are noted around the wrist joints, which are not calcified. There is no evidence of juxta-articular osteopenia. No osteophytes are noted. What diagnosis is most strongly suggested by these findings?

A. Gout.

B. CPPD.

C. RA.

D. Amyloidosis.

E. Wilson’s disease.

A
  1. D. Amyloidosis.

There are a lot of conditions that are capable of mimicking RA. In these cases a few key features can help reach a diagnosis. The classic finding in gout is of non-marginal erosions, as opposed to those described. Nevertheless, marginal erosions can occur with gout. An RA-type picture in the presence of non-marginal erosions or calcified soft-tissue nodules (tophi) should suggest this diagnosis. CPPD gives a more productive pattern of arthritis, such as seen with OA, affecting the radio-carpal joint. Thus, it is often suspected when the appearance is of OA with a ‘funny distribution’. Amyloidosis is suggested first by the history of MM. Involvement of the hands is more commonly seen in amyloid secondary to prolonged dialysis, but can be seen when the amyloid is secondary to MM, when the wrists are often affected. Amyloid can closely resemble RA in its distribution and the pattern of erosions. However, three important features can help differentiate: amyloidosis classically preserves the joint space, is not usually associated with periarticular osteopenia, and amyloidosis causes well-demarcated subchondral cyst formation in excess to that expected from the degree of joint disease.

69
Q
  1. A 35-year-old man presents with pain, swelling, and reduced movement of his knee. A plain film reveals a joint effusion, well-defined erosions with preservation of joint space, and normal bone mineralization. An MRI reveals, in addition, a mass in the region of the femoro-tibial joint space with low signal on T1WI and T2WI, and blooming artefact on GE imaging. What is the most likely diagnosis?

A. Synovial cell sarcoma.

B. Regional migratory osteoporosis.

C. Gout.

D. Synovial chondromatosis.

E. PVNS.

A
  1. E. PVNS.

This is a monoarticular tumour-like proliferation of synovium that occurs in joints, bursae, and tendon sheaths. It may be focal or diffuse. It occurs most frequently in the knee (80% of cases), then the hip, ankle, shoulder, and elbow. The abnormal synovium is prone to haemorrhage, thus producing blooming artefact on GE sequences, secondary to haemosiderin deposition.
In general the classic MRI appearance is variable low signal intensity on all sequences (T2WI signal being more variable due to fat, oedema, and blood products). Early changes involve a focal mass and joint effusion. Subsequently large erosions, synovial hypertrophy, and subchondral cysts may occur. Joint space is preserved until advanced disease is present and bone density is normal. After IV contrast at CT, PVNS shows variable enhancement, which can be striking. The differential diagnosis includes diseases causing recurrent haemarthroses, e.g. haemophilia and haemochromatosis (PVNS is monoarticular) as well as gout, amyloid, synovial chondromatosis, and tuberculosis.
Some 90% of synovial cell sarcomas do not originate from a joint. They are usually isointense to muscle on T1WI, with heterogeneous high-signal intensity on T2WI. Regional migratory osteoporosis would obviously involve loss of bone mineralization, as well as marrow oedema.
Gout demonstrates typically ‘rat’s bite’ para-articular erosions and soft-tissue calcification; when it involves the knee it tends to affect the patello-femoral compartment.

70
Q

2 A 72-year-old patient on haemodialysis presented with joint pain and effusion of the right shoulder. An arthroscopic specimen stained positive with Congo red. Plain films showed preserved joint space. Which is the most likely diagnosis?

a Rheumatoid arthritis

b Osteoarthritis

c Amyloid arthropathy

d Sarcoidosis

e Myeloma

A

2 Answer C: Amyloid arthropathy

Amyloid classically stains with Congo red and can be a primary or secondary process. Typical features include: bone pain, periarticular soft tissue swelling and carpal tunnel syndrome. Three patterns have been described: synovial articular pattern (amyloid arthropathy), diffuse marrow deposition and localised destructive lesion (amyloidoma, rarest form).

71
Q

17 A 37-year-old female with a long history of right knee pain is eventually diagnosed with pigmented villonodular synovitis (PVNS). What are the most likely findings on imaging?

a Marked joint space narrowing

b High signal of abnormal synovium on all MRI sequences

c Expansion of prefemoral fat pad

d Sclerotic deposits in articular surfaces

e joint effusion

A

17 Answer E: Joint effusion

The knee is the most common location for PVNS. The soft tissues appear dense due to haemosiderin deposits and there are multiple sites of cyst-like radiolucent defects due to bone invasion. There is not usually any evidence of calcification, osteoporosis and joint space narrowing until later on. There is low signal on all sequences due to the presence of haemosiderin. Other features include: knee joint effusion on plain films, scalloping of pre-femoral fat pad, soft-tissue mass around joint, no joint space narrowing.

72
Q

18 A 46-year-old coal miner with recently diagnosed rheumatoid arthritis was investigated for respiratory symptoms and found to have multiple rapidly developing lung nodules throughout the lungs with a upper and peripheral predominance. What complication is likely to have developed?

a Gaucher’s disease

b Caplan’s syndrome

C Felty’s syndrome

d Acute interstitial pneumonia (AIP)

e Panner’s disease

A

18 Answer B: Caplan’s syndrome

Caplan’s syndrome is a hyperimmune reactivity to silica inhalation with rapidly developing pulmonary nodules. Felty’s syndrome is RA plus splenomegaly and osteopenia. Panner’s disease is osteonecrosis of the capitellum.

73
Q

19 A 45-year-old woman with a history of a systemic disease presented with hand pain. What feature would favour a diagnosis of sarcoidosis over scleroderma?

a Reticulated trabecular pattern

b Calcinosis of soft tissues

C Sclerosis of terminal phalanges

d Soft-tissue swelling

e Bony erosions of carpal bones

A

19 Answer A: Reticulated trabecular pattern

Osseous involvement in sarcoid is uncommon, being reported in 6-20% of cases, and tends to affect the small bones of the hands and feet. The key features are a reticulated trabecular pattern, cyst-like lesions and destruction of the terminal phalanges. Other bony lesions are diffuse sclerosis of vertebral bodies and osteolytic changes in the skull.

74
Q

20 A 56-year-old female patient with stiffness and pain was referred for a radiograph of both hands. What features would favour a diagnosis of psoriatic arthritis over one of rheumatoid arthritis?

a Symmetrical distribution

b Predominant involvement of metacarpophalangeal joints

C Periarticular osteopenia

d Periosteal reaction

e Marginal erosions

A

20 Answer D: Periosteal reaction

Periosteal reaction is a frequent finding in psoriatic arthritis. Twenty per cent of patients with psoriasis develop psoriatic arthropathy which is classified into true psoriatic, resembling rheumatoid and concomitant rheumatoid and psoriatic type. The onset of arthritis often precedes the skin rash. The distribution is variable and asymmetrical, affecting hand and foot and axial skeleton. Features include `pencil in cup’ deformity, ivory phalanx, squaring of vertebrae and atlantoaxial subluxation.

75
Q

32 A 94-year-old lady was found on the floor in her nursing home. On arrival to the Emergency Department the attending doctor noticed a shortened and externally rotated right leg and radiographs of the right hip were requested. These show a minimally displaced fracture just below the right femoral neck and just above the greater and lesser trochanters. What is the treatment of choice?

a Dynamic hip screw

b Uncemented hemi-arthroplasty

C Cannulated screws

d Femoral head resurfacing

e Total hip replacement

A

32 Answer B: Uncemented hemi-arthroplasty

The treatment of choice for an intra-capsular (subcapital) femoral neck fracture in an elderly patient is hemi-arthroplasty. In a younger patient group, cannulated screws could be considered. Dynamic hip screws are indicated for extra-capsular fractures where there is little risk of avascular necrosis from femoral head devascularisation.

76
Q

(Ped) 58 A 12-month-old boy presents with pyrexia and a history of not moving his left leg for two days. On examination he is irritable and cries when the leg is manipulated. He has recently had an upper respiratory tract infection. Septic arthritis is suspected. Which of the following would be appropriate initial imaging?

a Bone scan of the hip

b Ultrasound of the ankle

c Ultrasound of the hip

d Plain radiograph of the knee

e CT of the knee

A

58 Answer C: Ultrasound of the hip

In children septic arthritis most commonly involves the hip, followed by the knee and then the ankle joint. Less frequently the site is the upper limb. Septic arthritis is usually associated with a joint effusion. Ultrasound is sensitive and specific for diagnosing hip effusions in children.

77
Q

67 A young adult female with severe acne had several episodes of inflammatory joint symptoms and a radiograph of her knee showed juxtaarticular osteoporosis. She also had evidence of osteosclerosis of several vertebrae with paravertebral ossification. What other skin rash is she at high risk of?

a Erythema nodosum

b Heliotrope rash

c Palmoplantar pustulosis

d Pemphigus

e Psoriasi

A

67 Answer C: Palmoplantar pustulosis

The patient has SAPHO syndrome (synovitis, acne, palmoplantar pustulosis, hyperostosis and osteitis).

78
Q

6 A 69-year-old female who had a right total knee replacement four years ago returns to clinic with knee pain. Her inflammatory markers and plain films are normal. A three-phase bone scan shows no increased uptake on dynamic or blood pool images, but static images show focal linear horizontal uptake in the medial tibial compartment adjacent to tibial component. Which is the most likely explanation?

a Early loosening of prosthesis

b Subclinical infection

c Weight-bearing load area of knee producing uptake

d Normal findings at this post-operative stage

e Referred pain from hip

A

6 Answer C: Weight-bearing load area of knee producing uptake

Bone scan appearances of knee replacements can take up to 18 months to normalise, as it is such a dynamic, weight-bearing joint. Loosening should be correlated with plain film findings such as adjacent bone lucency. The presence of normal inflammatory markers is reassuring, but subclinical infection should be considered with increased uptake in dynamic imaging. The hips and spine should be routinely imaged in lower limb pain to exclude referred pain. Load bearing changes on the medial aspect of the tibial plateau are not specific to joint replacements and in this case is the most likely cause of the pain.

79
Q

10 A 50-year-old man is known to have haemochromatosis and presents with pain in his hands. What distribution of involvement would be typical?

a Interphalangeal joint of thumb

b Proximal interphalangeal joints of all fingers

C Distal interphalangeal joints of middle and ring fingers

d Metacarpophalangeal joints of index and middle fingers

e Metacarpophalangeal joints of ring and little fingers

A

10 Answer E: Metacarpophalangeal joints of index and middle fingers

This classical distribution is distinctive although not every case will show the classical findings.

80
Q

10 A 50-year-old man is known to have haemochromatosis and presents with pain in his hands. What finding would be typical?

a Osteosclerosis

b Geodes

c Chondrocalcinosis

d Asymmetric joint space narrowing

e Narrowing of metacarpal heads

A

10 Answer C: Chondrocalcinosis

Haemochromatosis can be primary (inherited) or secondary (excessive iron absorption in anaemias, myelofibrosis, exogenous administration). Symptoms include: cirrhosis, congestive heart failure, arthritic symptoms. Skeletal features include: generalised osteoporosis, small subchondral cyst-like lesions in the metacarpal heads, arthropathy in 50%, uniform joint space narrowing, enlargement of metacarpal heads, osteophyte formation and chondrocalcinosis in 60%. The differential diagnosis includes: pseudogout, psoriatic arthritis, OA, RA, gout.

81
Q

11 A 55 year old with a history of several severe attacks of rheumatic fever presented with a polyarthropathy affecting the metacarpophalangeal joints of both hands with ulnar deviation and subluxation. The joint spaces were preserved and they were able to voluntarily correct the subluxation. What is the most likely diagnosis?

a Jaccoud’s arthropathy

b Rheumatoid arthropathy

C Amyloid arthropathy

d Psoriatic arthritis

e Haemophilic arthropathy

A

11 Answer A: Jaccoud’s arthropathy

This primarily affects the hands and occasionally the hallux. Radiographic features include muscular atrophy, periarticular swelling of the small joints of hands and feet, ulnar deviation and flexion of the MCP joints most marked in fourth and fifth finger without joint narrowing or erosion.

82
Q

13 A 56-year-old woman presented with an ankle deformity and was found to have reduced proprioception of this leg such that it was only minimally tender despite dislocation and marked bone destruction being visible on radiographs. What is a possible underlying cause?

a Charcot-Marie-Tooth

b Hyperthyroidism

C Prolonged use of antibiotics

d Acromegaly

e Use of hyoscine butylbromide (Buscopan®)

A

13 Answer A: Charcot-Marie-Tooth

Neuropathic osteoarthropathy is a traumatic arthritis associated with loss of sensation and proprioception of the affected limb. The causes can be congenital (Charcot-Marie-Tooth, Ehlers- Danlos), acquired (central/peripheral neuropathy, neurosyphilis, diabetes mellitus) or iatrogenic. Iatrogenic causes include: prolonged use of systemic or intra-articular steroids, and prolonged use of pain relieving drugs.

83
Q

16 A 67-year-old woman with known rheumatoid arthritis presented with right shoulder pain and a radiograph was performed. There was no history of trauma and secondary osteoarthritis was not thought to be a dominant feature. What features would be typical?

a Subchondral lucency

b Osteophytosis of the glenoid

C Increased joint space

d Widened glenohumeral joint space

e Tapered margin of distal clavicle

A

16 Answer E: Tapered margin of distal clavicle

Rheumatoid arthritis (RA) is a connective disease with associated immune complex deposition. There are early and late signs of the disease. RA affects hand and wrist, cervical spine, ribs, shoulder, hips, knee and feet. The main features are: erosions, loss of joint space, subchondral sclerosis. Other features include: osteoporosis, scalloped erosion on undersurface of clavicle, tapered margin of distal clavicle.

84
Q

18 A 50-year-old man presented with foot pain and radiographs showed involvement of a number of joints with generally non-specific findings. What underlying diagnosis would the presence of periostitis favour?

a Gout

b Pseudogout

C Psoriatic arthritis

d Haemophilia

e Rheumatoid arthritis

A

18 Answer C: Psoriatic arthritis

Other causes are juvenile rheumatoid arthritis, infectious arthritis and Reiter’s syndrome.

85
Q

(Ped) 65 A four-year-old boy presents with a three-week history of intermittent pyrexia. On examination he is miserable and febrile with a widespread salmon coloured rash, bilaterally enlarged lymph nodes and hepatosplenomegaly. Which of the following is the most likely diagnosis?

a Kawasaki disease

b Stevens Johnson syndrome

C Still’s disease

d Non-Hodgkin’s lymphoma

e Henoch-Schonlein purpura

A

65 Answer C: Still’s disease

This is a typical presentation of Still’s disease, systemic onset juvenile chronic arthritis. The arthritis may not be immediately apparent, but develops with time and persists once the systemic symptoms have resolved.

86
Q

68 A 37-year-old female presented with anterior knee pain and was investigated with an MRI scan. There was thinning of the cartilage in the patello-femoral joint, but no abnormality in the femorotibial joint. What is the most likely diagnosis?

a Septic arthritis

b Osgood- S chlatter disease

C Sinding-Larsen Johansson disease

d Chondromalacia patella

e Patella dislocation

A

68 Answer D: Chondromalacia patella

This common condition is due to softening of the articular cartilage and is often associated with patella maltracking. It is very often an asymptomatic finding at arthroscopy performed for another reason.

87
Q

A 20-year-old Caucasian male presented to his GP with backache. He was known to be HLA-B27 positive. Imaging of the sacroiliac (SI) joints showed bilateral symmetrical features, extensive sclerosis and erosions. Which of the following is the most likely diagnosis?

a Osteoarthritis

b Ankylosing spondylitis

C Osteitis condensans

d Reiter syndrome

e Infection

A

Answer B: Ankylosing spondylitis (AS)

This is the most likely diagnosis because it affects younger males, with extensive sclerosis and often with erosions. AS can cause both SI joint widening and fusion. Diseases that can cause bilateral symmetrical sacroiliitis include: AS, enteropathic arthropathy, late rheumatoid arthritis, deposition arthropathy and osteitis condensans ilii.

88
Q

8 A 74-year-old man with gout presented with recurrent foot pain and an X-ray of both his feet was taken. What is a typical feature of a joint affected by gout?

a joint space preserved until late in disease

b No joint effusion

c Periarticular demineralisation

d Erosions with thick sclerotic margins

e Chondrocalcinosis in the majority of cases

A

8 Answer A: Joint space preserved until late in disease

Gout is a disorder of purine metabolism, typically in middle-aged males, with four main parts: hyperuricaemia, deposition of positively birefringent crystals in synovial fluid, deposits of sodium urate in periarticular soft tissues, recurrent episodes of arthritis. It can be primary or secondary. The gouty tophus is pathognomonic histologically. The stages are: asymptomatic hyperuricaemia, acute gouty arthritis, chronic tophaceous gout and gouty nephropathy. Forty-five per cent of affected patients have radiological features, but these features are not seen until 6-12 years after the initial attack. Common locations are: joints, particularly hands and feet, bones, external ear. In the joint, effusion is the earliest sign, with periarticular swelling and preservation of joint space until late in the disease. The attacks are short so there is no periarticular demineralisation. Eccentric erosions are also seen, particularly at the metacarpal bases. Chondrocalcinosis is only seen in 5 %.

89
Q

13 A 60-year-old patient with a long history of diabetes peripheral neuropathy presented with ankle deformity. What feature would be in keeping with a Charcot joint?

a Juxta-articular osteoporosis

b Loose bodies

C Severe pain

d Blunted shape of metatarsal heads

e Widened joint space

A

13 Answer B: Loose bodies

The patient usually experiences a warm, painless joint, usually with a joint effusion, narrowed joint space, calcification in the soft tissues, and fragmentation of subchondral bone. Juxta-articular osteoporosis is not usually seen unless the joint is infected. Dense subchondral bone (sclerosis), Degeneration, Destruction of articular cortex, Deformity (‘pencil points’ of metatarsal heads), Debris and Dislocation is one way of helping remember the findings.

90
Q

14 A 14-year-old boy presents to his GP with joint pain, fever, rash and lymphadenopathy. Radiographs showed: rectangular phalanges in the hands and ribbon ribs, pleural effusion and pericardial effusion. What is the most likely diagnosis?

a Osteoarthritis

b Still disease

C Reiter’s disease

d Felty’s syndrome

e Psoriatic arthritis

A

14 Answer B: Still disease

This is essentially rheumatoid arthritis in patients younger than 16 and is more common in females. Still disease is systemic and may be polyarticular or pauci- articular. Typical findings include balloon epiphyses', gracile bones’, rectangular phalanges and ribbon ribs. Large joints tend to be involved first. Other differences from rheumatoid arthritis are a late onset of bony changes, more ankylosis and widening of the metaphyses.

91
Q

20 A 46-year-old man presented with back pain and was noted to have marked calcification and loss of height of multiple intervertebral discs, predominately in the lumbar spine. A previous radiograph of his knee also demonstrated premature osteoarthritic changes with pronounced chondrocalcinosis. What is the most likely diagnosis?

a Ankylosing spondylosis

b Ochronosis

C Hyperparathyroidism

d Wilson’s disease

e Reiter’s syndrome

A

20 Answer B: Ochronosis

Ochronosis or alkaptonuria is due to an inherited enzyme defect (homogentisic acid oxidase) and results in deposition of black pigment in cartilage. The features described are typical and affected individuals may also suffer from renal and cardiac failure.

92
Q

(Ped) 50 A nine-year-old girl presents with a two-month history of right knee swelling and morning stiffness with no definite history of trauma. On examination she is afebrile and the knee is swollen and tender with a reduced range of movement. She is thought to have juvenile rheumatoid arthritis. A knee radiograph is performed. Which of the following are the likely findings?

a Soft-tissue swelling and transverse metaphyseal bands

b Osteosclerosis and small epiphyses

C Soft tissue swelling, juxta-articular osteopenia and periosteal reaction

d Retarded bone growth and transverse metaphyseal bands

e Soft-tissue swelling and osteosclerosis

A

50 Answer C: Soft-tissue swelling, juxta-articular osteopenia and periosteal reaction

Juvenile rheumatoid arthritis commonly involves the large joints initially. Radiographic appearances later in the disease include erosions, epiphyseal overgrowth and ankylosis.

93
Q

64 A gentleman presented with early onset arthritis and generalised osteoporosis. His hands were worst affected with enlargement of the metacarpal head and loss of joint space particularly in the second and third metacarpophalangeal joints. Which malignancy is he at highest risk of?

a Non-small-cell lung cancer

b Testicular teratoma

C Hepatoma

d Colonic carcinoma

e Melanoma

A

64 Answer C: Hepatoma

The patient has haemochromatosis and cirrhosis is common. Cardiac failure is also a common cause of death. Females present later if at all due to the protective effect of menstruation.

94
Q

66 A 40-year-old woman presented with a long history of a painful joint and was found to have pigmented villonodular synovitis (PVNS). Which joint was most likely to have been affected?

a Ankle

b Elbow

c Hip

d Knee

e Shoulder

A

66 Answer D: Knee

Eighty per cent of cases occur in the knee and are usually monoarticular

95
Q
  1. A 40-year-old man presents with right knee pain. Plain radiography shows a large joint effusion. MRI of the knee shows multiple foci of low signal intensity seen in the synovium on T1, T2 and gradient-echo sequences. There is a moderate joint effusion. The most likely diagnosis is?

(a) Haemangioma

(b) Pigmented villonodular synovitis

(c) Rheumatoid arthritis

(d) Synovial sarcoma

(e) Synovial chondromatosis

A
  1. (b) Pigmented villonodular synovitis

This is a benign pathology affecting usually the knee joint. It shows no calcifications, osteoporosis or erosions (until late). MRI is diagnostic, the lesions returning low signal on all sequences due to iron (haemosiderin).

96
Q
  1. A 55-year-old man presents with pain and swelling in the left big toe. The plain radiograph shows periarticular erosions with sclerotic margins and overhanging edges in the first metatarsophalangeal joint. The joint space is preserved and there is moderate surrounding soft tissue swelling. The most likely diagnosis is?

(a) Rheumatoid arthritis

(b) Erosive osteoarthritis

(c) Gouty arthritis

(d) Psoriatic arthropathy

(e) Calcium pyrophosphate deposition disease.

A
  1. (c) Gouty arthritis

Periarticular erosions with sclerotic borders and overhanging margins with preserved articular surface are typical of gout.
Rheumatoid arthropathy has non-proliferative marginal erosions, symmetrical distribution and joint space narrowing. Psoriasis show progressive joint destruction with erosions. Erosive osteoarthritis is symmetrical with erosions are the articular surface Calcium pyrophosphate deposition disease is polyarticular, and shows chondrocalcinosis and joint-space narrowing.

97
Q
  1. An 80-year-old woman complains of pain in both hands. Radiography of the hands shows bilateral central articular ‘seagull’ erosions affecting the interphalangeal joints of fingers in both hands. Mild periarticular osteoporosis is seen. The most likely diagnosis is?

(a) Osteoarthritis

(b) Erosive osteoarthritis

(c) Calcium pyrophosphate deposition disease

(d) Gouty arthropathy

(e) Rheumatoid arthritis

A
  1. (b) Erosive osteoarthritis

Central articular erosion with a ‘seagull’ pattern, ankylosis and periarticular osteoporosis is typical of erosive osteoarthritis. This is seen in older women and is usually limited to hands, particularly affecting the proximal interphalangeal joints.

98
Q
  1. An 18-year-old man presents with progressive swelling of right knee. Radiographs show large joint effusion in the suprapatellar pouch. MRI shows marked synovial thickening and large synovial fronds which return high signal on T1, T2 and proton density images. The lesions are low signal on STIR images. The likely diagnosis is?

(a) Synovial lipoma

(b) Synovial osteochondromatosis

(c) Hypertrophic synovitis

(d) Pigmented villonodular synovitis

(e) Lipoma arborescens

A
  1. (e) Lipoma arborescens

This condition is seen most commonly in the suprapatellar pouch, with small to large frond-like masses arising from synovium. On MRI, the masses show characteristic signal of fat on all sequences. Saturation on STIR images is diagnostic.

99
Q
  1. A 15-year-old boy presents with pain and swelling in the hands. Radiographs show periarticular osteopenia, loss of joint space at the metacarpophalangeal joints and widened bases in the proximal phalanges. A periosteal reaction is seen in the metacarpal bones. What is the most likely diagnosis?

(a) Juvenile rheumatoid arthritis

(b) Psoriatic arthropathy

(c) Scleroderma

(d) Systemic lupus erythematosus

(e) Dermatomyositis

A
  1. (a) Juvenile rheumatoid arthritis

The condition is usually seen in young people before the age of 16 years. In the hand, the metacarpophalangeal and interphalangeal joints are usually affected. Chronic synovitis causes enlargement of bones and epiphyses. Malalignment and subluxation are common. An important feature of juvenile arthritis is periosteal reaction affecting the metacarpal and phalangeal shafts.

100
Q
  1. A 40-year-old immigrant from south Asia with a history of sexually transmitted disease treated 20 years ago presents with a painless, swollen right knee. Radiograph of the right knee shows collapse and fragmentation of the medial femoral condyle with subluxation of tibiofemoral joint. There is calcified debris in the knee and a large joint effusion. The bones show excessive sclerosis. What is the most likely diagnosis?

(a) Diabetes

(b) Haemophilia

(c) Charcot’s joint

(d) Calcium pyrophosphate deposition disease

(e) Osteoarthritis

A
  1. (c) Charcot’s joint

The radiographic features are those of Charcot’s joint. Given the history of treated sexually transmitted disease, syphilis must be considered.

101
Q
  1. A 70-year-old woman with a history of dysphagia presents with multiple swelling in the hands. Radiographs of the hands show widespread soft tissue calcification with terminal phalangeal resorption. What is the most likely diagnosis?

(a) Systemic lupus erythematosus

(b) Scleroderma

(c) Dermatomyositis

(d) Psoriasis

(e) Calcium pyrophosphate deposition disease

A
  1. (b) Scleroderma

Scleroderma is the cutaneous manifestation of progressive systemic sclerosis. This causes fibrosis and small vessel disease in several organs. In the hands, typically it causes terminal phalangeal resorption due to pressure atrophy, soft tissue calcification and occasionally intra-articular calcification.

102
Q
  1. A 40-year-old man presents with progressive pain and swelling of the left knee joint. MRI shows extensive low-signal synovial masses around the right knee on T1, T2 and STIR sequences. There is marked degenerative joint disease as well. What is the most likely diagnosis?

(a) Synovial chondromatosis

(b) Pigmented villonodular synovitis

(c) Synovial hypertrophy

(d) Lipoma arborescens

(e) Degenerative arthritis

A
  1. (b) Pigmented villonodular synovitis

The MRI appearances described are typical of pigmented villonodular synovitis, showing low signal of abnormal synovium on all sequences.

103
Q
  1. Typical feature of pigmented villonodular synovitis (PVNS) include: (T/F)

(a) Loss of bone density on plain film.

(b) A dense joint effusion on plain film.

(c) “blooming artefact” on gradient echo MR sequences.

(d) Well defined erosions with preservation of joint space.

(e) Soft tissue calcification on plain film.

A

Answer:

(a) Not correct
(b) Correct
(c) Correct
(d) Correct
(e) Not correct

Explanation:

PVNS represents part of a disease spectrum that includes diffuse and localized form of giant cell tumour of the tendon sheath to the more diffuse intra-articular form that is PVNS. Thus it is characterized by synovial proliferation and hemosiderin deposits. Hence, bone density is maintained and dense joint effusion is seen. Soft tissue calcification is uncommon in PVNS.

104
Q
  1. Which of the following are correct regarding ankylosing spondylitis: (T/F)

(a) The hip joints are involved in more than 20% of patients.

(b) 90 % of patients exhibit HLA-B27 antigen.

(c) Romanus lesions are seen in end-stage disease.

(d) Syndesmophytes are the result of ossification of the annulus fibrosus.

(e) Peripheral arthropathy is most common in the upper limbs.

A

Answer:

(a) Correct
(b) Correct
(c) Not correct
(d) Correct
(e) Not correct

Explanation:

Romanus lesions are seen in early stages of disease and consist of inflammation at the site of anterior longitudinal ligament attachment to vertebrae and discs giving rise to erosions at discovertebral junction. Extraspinal manifestations include enthesopathy and peripheral arthropathy favouring the lower limbs. Iritis, aortic insufficiency and pulmonary fibrosis are extraskeletal manifestations of Ankylosing spondylitis.

105
Q
  1. Which of the following are correct regarding synovial osteochondromatosis: (T/F)

(a) Most commonly affects the joints of the hands.

(b) Typically presents with a painless swollen joints.

(c) Intra-articular nodules can be hyper-intense on T2W.

(d) Calcification is absent in 25-30% of patients.

(e) Chondrosarcoma is a complication.

A

Answer:

(a) Not correct
(b) Not correct
(c) Correct
(d) Correct
(e) Correct

Explanation:

It is a benign monoarticular disorder of unknown etiology. It is more common in men usually presenting in 3rd-5th decade. Large joints like knee, hip, shoulder and elbow are commonly involved. However smaller joints like TM joint, AC joint and interphalangeal joint can also be involved. It presents with joint pain, swelling and restriction of movements.

106
Q
  1. Which of the following are correct regarding calcium pyrophosphate deposition disease (CPPD): (T/F)

(a) Haemochromatosis is a cause.

(b) Most commonly affects the small joints of the hand.

(c) Hyaline cartilage calcification is typically thick and irregular.

(d) Subchondral cyst formation is a typical feature.

(e) Destructive arthropathy is a rare manifestation.

A

Answer:

(a) Correct
(b) Not correct
(c) Not correct
(d) Correct
(e) Correct

Explanation:

CPPD associated acute arthritis is known as pseudogout. 50% cases affect the knee joint. Other locations are hip,shoulder, elbow and wrist joint. Changes are usually bilateral and symmetrical. Fibrocartilage calcification is typically coarse and irregular. Hyaline cartilage calcification is typically fine andlinear following the contour of underlying bone.

107
Q
  1. Features of rheumatoid arthritis include: (T/F)

(a) Late involvement of the 2nd and 3rd metacarpophalangeal joints

(b) Ulnar subluxation

(c) Central bone erosions

(d) Juxta-articular osteoporosis

(e) Calcification of the triangular fibrocartilage complex.

A

Answers:

(a) Not correct
(b) Correct
(c) Correct
(d) Correct
(e) Not correct

Explanation:

The metallic artefact is shows marginal and central erosions.
The second and third metacarpophalangeal joints are involved in early stage of the disease.
Calcification of triangular fibrocartilage complex is not usually seen in rheumatoid arthritis.